subject
Mathematics, 13.08.2019 05:10 Gipson

Aconvenience store manager noticed that a cooler which had been stocked with only a cola product and an iced-tea product had 15 colas left at the end of the day but only 3 iced-tea beverages. as a result, the manager reasoned that he should increase the amount of iced tea and decrease the amount of cola he ordered from the distributor. which of the following, if true, would most strengthen the manager's rationale for ordering more iced tea and less cola? a) the cooler in question is the only place in the store where the cola and iced tea beverages are stocked. b)on that day, a month-long $1,000,000 sweepstakes began, with prizes awarded via the bottlecaps on the iced tea beverage. c)at the beginning of the day, the cooler was stocked with at least as many of the iced tea beverages as of the cola beverages. d)on the subsequent day, the remaining three iced tea beverages all sold within the first hour after the store opened. e)during that week, a special "buy one, get one free" sale was in effect for the cola beverage.

ansver
Answers: 2

Another question on Mathematics

question
Mathematics, 21.06.2019 17:00
Can someone me i’m confused. will give brainliest and a whole bunch of points
Answers: 2
question
Mathematics, 21.06.2019 18:30
How do you create a data set with 8 points in it that has a mean of approximately 10 and a standard deviation of approximately 1?
Answers: 1
question
Mathematics, 21.06.2019 19:00
You are at a restaurant and owe $56.20 for your meal. you want to leave an 18% tip. approximately how much is your tip?
Answers: 1
question
Mathematics, 21.06.2019 19:20
Will give brainliest immeadeately true or false: the origin woud be included in the solution set for the inequality: y< 2x-5
Answers: 1
You know the right answer?
Aconvenience store manager noticed that a cooler which had been stocked with only a cola product and...
Questions
question
Mathematics, 20.11.2020 18:20
question
Health, 20.11.2020 18:20